Difference between revisions of "Aufgaben:Exercise 2.12: Non-coherent Demodulation"

From LNTwww
m
Line 3: Line 3:
 
}}
 
}}
  
[[File:P_ID1088__Mod_A_2_12.png|right|frame|Nichtkohärente <br>ASK–Demodulation]]
+
[[File:P_ID1088__Mod_A_2_12.png|right|frame|Non-coherent <br>ASK Demodulation]]
Wir betrachten ein AM–moduliertes Signal:
+
Consider an amplitude modulated signal:
 
:$$ s(t) = q(t)  \cdot \cos(\omega_{\rm T} \cdot t) \hspace{0.05cm}.$$
 
:$$ s(t) = q(t)  \cdot \cos(\omega_{\rm T} \cdot t) \hspace{0.05cm}.$$
Den Empfänger erreicht aufgrund der Kanallaufzeit das Signal
+
Reaching the receiver based on the channel propagation time, the signal is
 
:$$ r(t) = q(t) \cdot \cos(\omega_{\rm T} \cdot t + \Delta \phi_{\rm T}) \hspace{0.05cm}.$$
 
:$$ r(t) = q(t) \cdot \cos(\omega_{\rm T} \cdot t + \Delta \phi_{\rm T}) \hspace{0.05cm}.$$
Die skizzierte Anordnung erlaubt eine perfekte Demodulation das heißt &nbsp;$v(t) = q(t)$ – ohne Kenntnis der Phase &nbsp;$Δϕ_T$, allerdings nur dann, wenn das Quellensignal &nbsp;$q(t)$&nbsp; gewisse Voraussetzungen erfüllt.
+
The arrangement shown here allows perfect demodulation that is, &nbsp;$v(t) = q(t)$ – without knowledge of the phase &nbsp;$Δϕ_T$, but only if the source signal &nbsp;$q(t)$&nbsp; satisfies certain conditions.
  
Die beiden empfängerseitigen Trägersignale lauten:
+
The two receiver-side carrier signals are:
 
:$$ z_{\rm 1, \hspace{0.08cm}E}(t)  =  2 \cdot \cos(\omega_{\rm T} \cdot t) \hspace{0.05cm},$$
 
:$$ z_{\rm 1, \hspace{0.08cm}E}(t)  =  2 \cdot \cos(\omega_{\rm T} \cdot t) \hspace{0.05cm},$$
 
:$$ z_{\rm 2, \hspace{0.08cm}E}(t)  =  -2 \cdot \sin(\omega_{\rm T} \cdot t) \hspace{0.05cm}.$$
 
:$$ z_{\rm 2, \hspace{0.08cm}E}(t)  =  -2 \cdot \sin(\omega_{\rm T} \cdot t) \hspace{0.05cm}.$$
  
$\rm TP_1$&nbsp; und &nbsp;$\rm TP_2$&nbsp; bezeichnen zwei ideale (rechteckförmige) Tiefpässe, deren Grenzfrequenz jeweils gleich der Trägerfrequenz &nbsp;$f_{\rm T}$&nbsp; ist.&nbsp;
+
$\rm TP_1$&nbsp; and &nbsp;$\rm TP_2$&nbsp; denote two ideal (rextangular) lowpass filters, each with cutoff frequency equal to the carrier frequency &nbsp;$f_{\rm T}$&nbsp;.
  
Als (digitale) Quellensignale werden betrachtet:
+
As (digital) source signals we consider:
* das unipolare Rechtecksgnal &nbsp;$q_1(t)$&nbsp; mit den dimensionslosen Amplitudenwerten &nbsp;$0$&nbsp; und &nbsp;$3$,
+
* the unipolar square wave &nbsp;$q_1(t)$&nbsp; with dimensionless amplitude values &nbsp;$0$&nbsp; and &nbsp;$3$,
* das bipolare Rechtecksignal &nbsp;$q_2(t)$&nbsp; mit den dimensionslosen Amplitudenwerten &nbsp;$±3$.
+
* the bipolar square wave signal &nbsp;$q_2(t)$&nbsp; with the dimensionless amplitude values &nbsp;$±3$.
  
  
Diese beiden Signale ergeben hinsichtlich &nbsp;$s(t)$&nbsp; ein &nbsp;[[Modulation_Methods/Lineare_digitale_Modulationsverfahren#ASK_.E2.80.93_Amplitude_Shift_Keying|ASK–Signal]]&nbsp; bzw. ein &nbsp;[[Modulation_Methods/Lineare_digitale_Modulationsverfahren#BPSK_.E2.80.93_Binary_Phase_Shift_Keying|BPSK–Signal]].
+
With respect to &nbsp;$s(t)$&nbsp;, these two signals result in an &nbsp;[[Modulation_Methods/Linear_Digital_Modulation#ASK_.E2.80.93_Amplitude_Shift_Keying|ASK signal]]&nbsp; and a &nbsp;[[Modulation_Methods/Linear_Digital_Modulation#BPSK_.E2.80.93_Binary_Phase_Shift_Keying|BPSK signal]], respectively.
  
Die nichtlineare Funktion &nbsp;$v = g(b)$&nbsp; soll im Rahmen dieser Aufgabe ermittelt werden.
+
The nonlinear function &nbsp;$v = g(b)$&nbsp; is to be determined in this exercise.
  
  
Line 32: Line 32:
  
  
''Hinweise:''  
+
''Hints:''  
*Die Aufgabe gehört zum  Kapitel&nbsp; [[Modulation_Methods/Weitere_AM–Varianten|Weitere AM–Variantenn]].
+
*This exercise belongs to the chapter &nbsp; [[Modulation_Methods/Further_AM_Variants|Further AM Variants]].
*Bezug genommen wird insbesondere auf die Seite&nbsp;  [[Modulation_Methods/Weitere_AM–Varianten#Inkoh.C3.A4rente_.28nichtkoh.C3.A4rente.29_Demodulation|Inkohärente (nichtkohärente) Demodulation]].
+
*Particular reference is made to the page&nbsp;  [[Modulation_Methods/Further_AM_Variants#Incoherent_.28non-coherent.29_Demodulation|Incoherent (non-coherent) Demodulation]].
 
   
 
   
*Gegeben sind folgende trigonometrischen Umformungen:
+
*The following trigonometric transformations are given:
 
:$$ \cos(\alpha) \cdot \cos(\beta)  = 1/2 \cdot \big[ \cos(\alpha - \beta)+ \cos(\alpha + \beta) \big],$$
 
:$$ \cos(\alpha) \cdot \cos(\beta)  = 1/2 \cdot \big[ \cos(\alpha - \beta)+ \cos(\alpha + \beta) \big],$$
 
:$$ \sin(\alpha) \cdot \sin(\beta)  = 1/2 \cdot \big[ \cos(\alpha - \beta)- \cos(\alpha + \beta) \big],$$
 
:$$ \sin(\alpha) \cdot \sin(\beta)  = 1/2 \cdot \big[ \cos(\alpha - \beta)- \cos(\alpha + \beta) \big],$$
Line 43: Line 43:
  
  
===Fragebogen===
+
===Questions===
  
 
<quiz display=simple>
 
<quiz display=simple>
{Wie lauten die Signale &nbsp;$b_1(t)$&nbsp; und &nbsp;$b_2(t)$&nbsp; in den beiden Zweigen jeweils nach Multiplizierer und Tiefpass?&nbsp; Welche Aussagen treffen zu?
+
{What are the signals &nbsp;$b_1(t)$&nbsp; and &nbsp;$b_2(t)$&nbsp; in both branches after multiplier and lowpass respectively?&nbsp; Which statements apply?
 
|type="[]"}
 
|type="[]"}
 
+ $b_1(t) = q(t) · \cos(Δϕ_{\rm T})$.
 
+ $b_1(t) = q(t) · \cos(Δϕ_{\rm T})$.

Revision as of 21:46, 22 December 2021

Non-coherent
ASK Demodulation

Consider an amplitude modulated signal:

$$ s(t) = q(t) \cdot \cos(\omega_{\rm T} \cdot t) \hspace{0.05cm}.$$

Reaching the receiver based on the channel propagation time, the signal is

$$ r(t) = q(t) \cdot \cos(\omega_{\rm T} \cdot t + \Delta \phi_{\rm T}) \hspace{0.05cm}.$$

The arrangement shown here allows perfect demodulation – that is,  $v(t) = q(t)$ – without knowledge of the phase  $Δϕ_T$, but only if the source signal  $q(t)$  satisfies certain conditions.

The two receiver-side carrier signals are:

$$ z_{\rm 1, \hspace{0.08cm}E}(t) = 2 \cdot \cos(\omega_{\rm T} \cdot t) \hspace{0.05cm},$$
$$ z_{\rm 2, \hspace{0.08cm}E}(t) = -2 \cdot \sin(\omega_{\rm T} \cdot t) \hspace{0.05cm}.$$

$\rm TP_1$  and  $\rm TP_2$  denote two ideal (rextangular) lowpass filters, each with cutoff frequency equal to the carrier frequency  $f_{\rm T}$ .

As (digital) source signals we consider:

  • the unipolar square wave  $q_1(t)$  with dimensionless amplitude values  $0$  and  $3$,
  • the bipolar square wave signal  $q_2(t)$  with the dimensionless amplitude values  $±3$.


With respect to  $s(t)$ , these two signals result in an  ASK signal  and a  BPSK signal, respectively.

The nonlinear function  $v = g(b)$  is to be determined in this exercise.





Hints:

  • The following trigonometric transformations are given:
$$ \cos(\alpha) \cdot \cos(\beta) = 1/2 \cdot \big[ \cos(\alpha - \beta)+ \cos(\alpha + \beta) \big],$$
$$ \sin(\alpha) \cdot \sin(\beta) = 1/2 \cdot \big[ \cos(\alpha - \beta)- \cos(\alpha + \beta) \big],$$
$$ \sin(\alpha) \cdot \cos(\beta) = 1/2 \cdot \big[ \sin(\alpha - \beta)+ \sin(\alpha + \beta) \big] \hspace{0.05cm}.$$


Questions

1

What are the signals  $b_1(t)$  and  $b_2(t)$  in both branches – after multiplier and lowpass respectively?  Which statements apply?

$b_1(t) = q(t) · \cos(Δϕ_{\rm T})$.
$b_2(t) = q(t) · \cos(Δϕ_{\rm T})$.
$b_1(t) = q(t) · \sin(Δϕ_{\rm T})$.
$b_2(t) = q(t) · \sin(Δϕ_{\rm T})$.
$b_1(t) = b_2(t) = q(t)$.

2

Welche Werte  $b_{\rm min}$  und  $b_{\rm max}$  nimmt das Signal  $b(t)$  an, wenn am Eingang das unipolare Quellensignal  $q_1(t)$  anliegt?

$b_{\rm min} \ = \ $

$b_{\rm max} \ = \ $

3

Wie muss die Kennlinie  $v = g(b)$  gewählt werden, damit  $v(t) = q(t)$  gilt?

$v=g(b) = b^2$.
$v=g(b) = \sqrt{b}$.
$v=g(b) = \arctan(b).$

4

Welche Werte  $b_{\rm min}$  und  $b_{\rm max}$  nimmt das Signal  $b(t)$  an, wenn am Eingang das bipolare Quellensignal  $q_2(t)$  anliegt?

$b_{\rm min} \ = \ $

$b_{\rm max} \ = \ $


Musterlösung

(1)  Durch Anwendung der auf der Angabenseite gegebenen trigonometrischen Umformungen erhält man unter Berücksichtigung der beiden Tiefpässe (die Anteile um die doppelte Trägerfrequenz werden entfernt):

$$b_1(t) = q(t) \cdot \cos(\omega_{\rm T} \cdot t + \Delta \phi_{\rm T}) \cdot 2 \cdot \cos(\omega_{\rm T} \cdot t) = q(t) \cdot \cos(\Delta \phi_{\rm T})\hspace{0.05cm},$$
$$ b_2(t) = q(t) \cdot \cos(\omega_{\rm T} \cdot t + \Delta \phi_{\rm T}) \cdot (-2) \cdot \sin(\omega_{\rm T} \cdot t) = q(t) \cdot \sin(\Delta \phi_{\rm T})\hspace{0.05cm}.$$
  • Richtig sind somit die erste und die vierte Antwort.


(2)  Die Summe der Quadrate der beiden Teilsignale ergibt:

$$ b(t) = b_1^2(t) + b_2^2(t)= q^2(t) \cdot \left( \cos^2(\Delta \phi_{\rm T})+ \sin^2(\Delta \phi_{\rm T})\right) = q^2(t)\hspace{0.05cm}.$$

Die möglichen Amplitudenwerte sind somit:  

$$b_{\rm min}\hspace{0.15cm}\underline{ = 0},$$
$$ b_{\rm max}\hspace{0.15cm}\underline{ =9}.$$


(3)  Richtig ist der zweite Lösungsvorschlag:

$$v=g(b) = \sqrt{b} \hspace{0.3cm} \Rightarrow \hspace{0.3cm} v(t) = \sqrt{ q^2(t) } = q(t)\hspace{0.05cm}.$$


(4)  Das Ergebnis  $b(t) = q^2(t)$ – siehe Teilaufgabe  (2)  – führt hier zum Ergebnis:

$$b_{\rm min}\hspace{0.15cm}\underline{ = 9},$$
$$b_{\rm max}\hspace{0.15cm}\underline{ =9}.$$

Dies zeigt, dass der hier betrachtete Demodulator nur dann funktioniert, wenn für alle Zeiten  $q(t) ≥ 0$  oder  $q(t) ≤ 0$  gilt und dies dem Empfänger auch bekannt ist.